Paramagnetismo Spin-1/2 Partículas - Función de partición

Estoy tratando de encontrar una expresión para la función de partición de un sistema de partículas de gas ideal spin-1/2 en una línea de longitud L . El número total de partículas norte es fijo, con norte = norte + norte . Aquí, norte es el número de partículas spin-up y norte es el número de partículas spin-down en un microestado particular.

Tengo el siguiente hamiltoniano para las partículas de masa metro .

H = i = 1 norte ( pag i + β s i ) 2 2 metro b s i

Aquí, s i = 1 para el giro norte partículas y s i = 1 para el spin-down norte partículas β y b son constantes.

Estoy tratando de usar el hamiltoniano para escribir la energía de las partículas que giran hacia arriba y hacia abajo para poder escribir la función de partición. Si expando el hamiltoniano, obtengo:

H = i = 1 norte pag i 2 2 metro + pag i β s i metro b s i + β 2

¿Cómo encuentro la energía de los dos conjuntos de partículas de espín a partir de esto y la uso para generar la función de partición? ¿Es justa la energía de las partículas?

mi = pag i 2 2 metro + pag i β metro b + β 2
mi = pag i 2 2 metro + pag i β metro + b + β 2

¿Cómo se evaluaría esto en la función de partición canónica?

Z = m s mi β H

dónde m s es la suma de todos los microestados. No estoy seguro de cómo evaluar esto.

¿Estás descuidando la interacción magnética entre partículas, H = i , j j i j s i s j ?
No, no tengo este término en el hamiltoniano. Solo estoy tratando de tratar el movimiento orbital de las partículas de forma clásica.

Respuestas (1)

En primer lugar, escriba una expresión explícita para la suma de todos los microestados.

Editar Dado que está tratando el sistema de forma clásica, esto incluye una integral sobre el espacio de fase y una suma sobre todas las configuraciones de espín posibles.

m s = { s i } d norte pag d norte q h 3 norte norte !

Lo segundo es darse cuenta de que su hamiltoniano no interactúa y la densidad canónica mi β H es solo un producto de hamiltonianos de una partícula

mi β H = i = 1 norte mi β h i
donde por supuesto
h i = ( pag i + γ s i ) 2 2 metro b s i
(Cambié el nombre β a γ , porque no te refieres a la temperatura inversa. aquí)

Entonces hay que evaluar

1 norte ! { s i } i = 1 norte d pag i d q i h 3 mi β h i =

Porque H Como no interactúa, la integral de espacio de fase de N partículas se factoriza en N integraciones sobre un espacio de fase de 1 partícula. De manera similar, uno puede intercambiar la suma de espín y el producto (¡convénzase de que esto es cierto! Por ejemplo, uno termina con los mismos términos, ya sea que sume sobre el espín primero o no).

Eso significa que, en lugar de sumar todos los microestados de muchos cuerpos, uno primero suma las posibles configuraciones de una sola partícula y explica el hecho de que hay muchos después. Además todos h i son equivalentes. Cada uno de ellos lleva un índice diferente pero redundante:

Z = 1 norte ! i = 1 norte s i = ± 1 d pag i d q i h 3 mi β h i = 1 norte ! ( s = ± 1 d pag d q h 3 mi β h ) norte
dónde h es h i pero sin índice, porque la referencia ya no es a una partícula específica.

/Editar

Tendrás que pensar qué hacer con la integración de momento. No he calculado el resultado, pero es posible que no termine con una solución en forma cerrada. Es posible que se necesite una aproximación para hacer la suma del impulso. Editar Creo que una integración gaussiana hará el truco. /Editar ¡ Cuéntanos con qué terminas!

@ vectorize7891 ¿Hizo algún progreso? ¿Necesita más aclaraciones?
¡Muchas gracias por dar seguimiento a esto! He hecho algun progreso. ¿No necesita también sumar posiciones? La única forma en que podría evaluar esto es para grandes norte . Otra preocupación que tenía es, para grandes norte , no necesariamente sabes eso norte Y norte ambos serán grandes, ¿verdad?
@ vectorize7891 Tiene razón con respecto a la integración de volumen. He actualizado mi respuesta. Creo que bastará con considerar grandes norte en algún momento, sin hacer referencia a norte o norte . ¿Dónde entran durante su cálculo? Aunque en el límite del campo externo cero, sus valores esperados serán norte / 2 , un campo externo los separará.